LSAT and Law School Admissions Forum

Get expert LSAT preparation and law school admissions advice from PowerScore Test Preparation.

 amgard
  • Posts: 1
  • Joined: Nov 08, 2019
|
#71836
Can you explain why C is the correct answer?
 Jeremy Press
PowerScore Staff
  • PowerScore Staff
  • Posts: 1000
  • Joined: Jun 12, 2017
|
#71885
Hi amgard,

Sure! This is a Parallel Flaw question, meaning we need to identify an answer choice with an argument that exhibits the same logical flaw (or flaws) as the argument in the stimulus.

The argument in the stimulus contains a species of what we call the Mistaken Negation flaw. The first sentence states a conditional relationship: if a book presents a certain kind of utopian future, then it will find enthusiastic buyers. The necessary condition in that relationship is "find enthusiastic buyers," and the sufficient condition is that a book "present a utopian future." Under the principles of conditional reasoning, a necessary condition is something that can occur with or without its sufficient condition, and without additional clarifying information, we won't know how likely that necessary condition is to occur in the absence of the sufficient condition. But, the author of the argument has concluded that the necessary condition is unlikely to occur, purely on the basis of the absence of the sufficient condition. This is, in essence, the Mistaken Negation flaw. Notice one other thing: the true necessary condition in the first sentence is "find enthusiastic buyers," the true logical negation of which would be "not find enthusiastic buyers." The conclusion states something that is more like a polar opposite: "unlikely to be very popular."

Answer choice C exhibits both of these issues. The first premise states a conditional relationship: if an action movie has certain kinds of special-effects scenes, then it is enormously expensive to produce. The necessary condition is "enormously expensive to produce," and the sufficient condition is "complicated and dangerous special-effects scenes." The author then concludes that the (polar, not logical) opposite of the necessary condition is probable, purely on the basis of the absence of the sufficient condition. It's another Mistaken Negation, with the twist that the conclusion speaks to the extreme opposite of the necessary condition, rather than the true logical opposite of the necessary condition (which would be "not enormously expensive to produce").

Let us know if this clears things up!

Jeremy

Get the most out of your LSAT Prep Plus subscription.

Analyze and track your performance with our Testing and Analytics Package.